Last visit was: 25 Apr 2024, 06:54 It is currently 25 Apr 2024, 06:54

Close
GMAT Club Daily Prep
Thank you for using the timer - this advanced tool can estimate your performance and suggest more practice questions. We have subscribed you to Daily Prep Questions via email.

Customized
for You

we will pick new questions that match your level based on your Timer History

Track
Your Progress

every week, we’ll send you an estimated GMAT score based on your performance

Practice
Pays

we will pick new questions that match your level based on your Timer History
Not interested in getting valuable practice questions and articles delivered to your email? No problem, unsubscribe here.
Close
Request Expert Reply
Confirm Cancel
SORT BY:
Date
Tags:
Show Tags
Hide Tags
User avatar
Manager
Manager
Joined: 11 Feb 2011
Posts: 86
Own Kudos [?]: 1091 [9]
Given Kudos: 21
Send PM
Most Helpful Reply
GMAT Club Legend
GMAT Club Legend
Joined: 12 Sep 2015
Posts: 6821
Own Kudos [?]: 29925 [3]
Given Kudos: 799
Location: Canada
Send PM
General Discussion
User avatar
Retired Moderator
Joined: 20 Dec 2010
Posts: 1114
Own Kudos [?]: 4702 [0]
Given Kudos: 376
Send PM
Retired Moderator
Joined: 19 Mar 2014
Posts: 817
Own Kudos [?]: 969 [0]
Given Kudos: 199
Location: India
Concentration: Finance, Entrepreneurship
GPA: 3.5
Send PM
Re: If xy > 0, which of the following CANNOT be true? [#permalink]
Easy question to solve, should not take more than 1 min of your time.

xy>0

Which means either:

x & y are +ve
x & y are -ve

Let check the options now.

A. x>0 - Possible (as x and y can have +ve values)
B. y<0 - Possible (as x and y can have -ve values)
C. x+y>0 - possible (as x and y can have +values and hence addition will give as a +ve number)
D. x/y>0 - possible (as x and y can have +values and hence division will give as a +ve number)
E. x/y<0 - NOT POSSIBLE (as x & y can both be +ve - which gives us +ve number post division and x & y can be -ve which again gives us positive number post division)

Hence, Answer is E
Manager
Manager
Joined: 03 Feb 2017
Posts: 69
Own Kudos [?]: 104 [0]
Given Kudos: 31
Location: Australia
GMAT 1: 720 Q48 V40
Send PM
Re: If xy > 0, which of the following CANNOT be true? [#permalink]
Since xy>0, it means either both x and y are positive or both are negative.
However, according to E, only one term is positive, so E is not possible and hence the answer
Manager
Manager
Joined: 17 Feb 2016
Posts: 71
Own Kudos [?]: 151 [0]
Given Kudos: 59
Location: India
Concentration: General Management, Entrepreneurship
GMAT 1: 660 Q47 V36
GPA: 3.12
WE:Education (Internet and New Media)
Send PM
Re: If xy > 0, which of the following CANNOT be true? [#permalink]
xy>0
This implies the product is positive
X and Y can have only same signs

X,Y will be either (-ve,-ve) or (+ve,+ve)

Hence, \(\frac{x}{y}\)= cannot be negative



Alternatively, One can assume a value for the product and check for the possible conditions
For example

xv=6 (which is >0)
Possible values of x,y are (-3,-2), (3,2), (-1/6,-36), (1/6,36)
Substitute any values in the options, Option E alone will not be possible
Target Test Prep Representative
Joined: 04 Mar 2011
Status:Head GMAT Instructor
Affiliations: Target Test Prep
Posts: 3043
Own Kudos [?]: 6273 [0]
Given Kudos: 1646
Send PM
Re: If xy > 0, which of the following CANNOT be true? [#permalink]
Expert Reply
AnkitK wrote:
If xy>0, which of the foll cannot be true?

A. x>0
B. y<0
C. x+y>0
D. x/y>0
E. x/y<0


Since xy > 0, x and y have the same sign. We see that x and y are either both positive or both negative. Thus, x/y could never be negative.

Answer: E
Director
Director
Joined: 20 Dec 2015
Status:Learning
Posts: 876
Own Kudos [?]: 566 [0]
Given Kudos: 755
Location: India
Concentration: Operations, Marketing
GMAT 1: 670 Q48 V36
GRE 1: Q157 V157
GPA: 3.4
WE:Engineering (Manufacturing)
Send PM
Re: If xy > 0, which of the following CANNOT be true? [#permalink]
imo e
x and y will have same sign hence e is not possible .
User avatar
Non-Human User
Joined: 09 Sep 2013
Posts: 32673
Own Kudos [?]: 821 [0]
Given Kudos: 0
Send PM
Re: If xy > 0, which of the following CANNOT be true? [#permalink]
Hello from the GMAT Club BumpBot!

Thanks to another GMAT Club member, I have just discovered this valuable topic, yet it had no discussion for over a year. I am now bumping it up - doing my job. I think you may find it valuable (esp those replies with Kudos).

Want to see all other topics I dig out? Follow me (click follow button on profile). You will receive a summary of all topics I bump in your profile area as well as via email.
GMAT Club Bot
Re: If xy > 0, which of the following CANNOT be true? [#permalink]
Moderators:
Math Expert
92912 posts
Senior Moderator - Masters Forum
3137 posts

Powered by phpBB © phpBB Group | Emoji artwork provided by EmojiOne